Difference between revisions of "2001 AIME I Problems/Problem 9"

Line 32: Line 32:
  
 
By the barycentric area formula, our desired ratio is equal to  
 
By the barycentric area formula, our desired ratio is equal to  
<cmath>\det A=
+
<cmath>\begin{align*}
 
\begin{vmatrix}
 
\begin{vmatrix}
 
1-p & p & 0 \\  
 
1-p & p & 0 \\  
 
0 & 1-q & q  \\  
 
0 & 1-q & q  \\  
 
r & 0 & 1-r \notag
 
r & 0 & 1-r \notag
\end{vmatrix} =1-p-q-r+pq+qr+pr=1-(p+q+r)+\frac{(p+q+r)^2-(pq+qr-pr)}{2}=1-\frac{2}{3}+\frac{\frac{4}{9}-\frac{2}{5}}{2}=\frac{16}{45},</cmath> so the answer is <math>\boxed{61.}</math>
+
\end{vmatrix} &=1-p-q-r+pq+qr+pr\\
 +
&=1-(p+q+r)+\frac{(p+q+r)^2-(p^2+q^2+r^2)}{2}\\
 +
&=1-\frac{2}{3}+\frac{\frac{4}{9}-\frac{2}{5}}{2}\\
 +
&=\frac{16}{45}
 +
\end{align*},</cmath> so the answer is <math>\boxed{61.}</math>
 
== See also ==
 
== See also ==
 
{{AIME box|year=2001|n=I|num-b=8|num-a=10}}
 
{{AIME box|year=2001|n=I|num-b=8|num-a=10}}

Revision as of 22:05, 27 May 2016

Problem

In triangle $ABC$, $AB=13$, $BC=15$ and $CA=17$. Point $D$ is on $\overline{AB}$, $E$ is on $\overline{BC}$, and $F$ is on $\overline{CA}$. Let $AD=p\cdot AB$, $BE=q\cdot BC$, and $CF=r\cdot CA$, where $p$, $q$, and $r$ are positive and satisfy $p+q+r=2/3$ and $p^2+q^2+r^2=2/5$. The ratio of the area of triangle $DEF$ to the area of triangle $ABC$ can be written in the form $m/n$, where $m$ and $n$ are relatively prime positive integers. Find $m+n$.

Solution

Solution 1

[asy]  /* -- arbitrary values, I couldn't find nice values for pqr please replace if possible -- */  real p = 0.5, q = 0.1, r = 0.05;   /* -- arbitrary values, I couldn't find nice values for pqr please replace if possible -- */  pointpen = black; pathpen = linewidth(0.7) + black; pair A=(0,0),B=(13,0),C=IP(CR(A,17),CR(B,15)), D=A+p*(B-A), E=B+q*(C-B), F=C+r*(A-C); D(D(MP("A",A))--D(MP("B",B))--D(MP("C",C,N))--cycle); D(D(MP("D",D))--D(MP("E",E,NE))--D(MP("F",F,NW))--cycle); [/asy]

We let $[\ldots]$ denote area; then the desired value is

$\frac mn = \frac{[DEF]}{[ABC]} = \frac{[ABC] - [ADF] - [BDE] - [CEF]}{[ABC]}$

Using the formula for the area of a triangle $\frac{1}{2}ab\sin C$, we find that

$\frac{[ADF]}{[ABC]} = \frac{\frac 12 \cdot p \cdot AB \cdot (1-r) \cdot AC \cdot \sin \angle CAB}{\frac 12 \cdot AB \cdot AC \cdot \sin \angle CAB} = p(1-r)$

and similarly that $\frac{[BDE]}{[ABC]} = q(1-p)$ and $\frac{[CEF]}{[ABC]} = r(1-q)$. Thus, we wish to find \begin{align*}\frac{[DEF]}{[ABC]} &= 1 - \frac{[ADF]}{[ABC]} - \frac{[DEF]}{[BDE]} - \frac{[CEF]}{[ABC]}  \\ &= 1 - p(1-r) + q(1-p) + r(1-q)\\ &= (pq + qr + rp) - (p + q + r) + 1 \end{align*} We know that $p + q + r = \frac 23$, and also that $(p+q+r)^2 = p^2 + q^2 + r^2 + 2(pq + qr + rp) \Longleftrightarrow pq + qr + rp = \frac{\left(\frac 23\right)^2 - \frac 25}{2} = \frac{1}{45}$. Substituting, the answer is $\frac 1{45} - \frac 23 + 1 = \frac{16}{45}$, and $m+n = \boxed{061}$.

Solution 2

By the barycentric area formula, our desired ratio is equal to \begin{align*} \begin{vmatrix} 1-p & p & 0 \\  0 & 1-q & q  \\  r & 0 & 1-r \notag \end{vmatrix} &=1-p-q-r+pq+qr+pr\\ &=1-(p+q+r)+\frac{(p+q+r)^2-(p^2+q^2+r^2)}{2}\\ &=1-\frac{2}{3}+\frac{\frac{4}{9}-\frac{2}{5}}{2}\\ &=\frac{16}{45} \end{align*}, so the answer is $\boxed{61.}$

See also

2001 AIME I (ProblemsAnswer KeyResources)
Preceded by
Problem 8
Followed by
Problem 10
1 2 3 4 5 6 7 8 9 10 11 12 13 14 15
All AIME Problems and Solutions

The problems on this page are copyrighted by the Mathematical Association of America's American Mathematics Competitions. AMC logo.png